fai la somma

Il forum di Base5, dove è possibile postare problemi, quiz, indovinelli, rompicapo, enigmi e quant'altro riguardi la matematica ricreativa e oltre.

Moderatori: Gianfranco, Bruno

Rispondi
Pasquale
Livello 12
Livello 12
Messaggi: 2853
Iscritto il: mer mag 25, 2005 2:14 am

fai la somma

Messaggio da Pasquale »

ANNULLATO

Anche di questo non ho la soluzione, ma ve lo passo per la vostra goduria (mi sa che deve essere un po' tosto):

trovare la somma della serie infinita $1+\frac{1}{2}+\frac{1}{3}+\frac{1}{4}+\frac{1}{6}+\frac{1}{12}+ ....….$, in cui al denominatore compaiono solo numeri divisibili per 2 e/o 3 (giustificare).

ANNULLATO
Ultima modifica di Pasquale il mer dic 28, 2005 6:51 pm, modificato 1 volta in totale.
_________________

$\text { }$ciao Immagine ciao
E' la somma che fa il totale (Totò)

Bruno
Livello 10
Livello 10
Messaggi: 2020
Iscritto il: lun nov 21, 2005 6:07 pm
Località: Bologna

Messaggio da Bruno »

...

Pasquale, ciao.

Ho bisogno di un chiarimento, mi sfugge qualcosa.
Il quattro e il due non son divisibili per tre, perché allora non c'è l'otto?
E perché non c'è il nove ma il tre sì?
I denominatori devono essere della forma $\displaystyle {\tex\footnotesize 2^\alpha \cdot 3^\beta}$ con esponenti non negativi?
Intanto grazie ;)

Bruno

Pasquale
Livello 12
Livello 12
Messaggi: 2853
Iscritto il: mer mag 25, 2005 2:14 am

Messaggio da Pasquale »

...e il 10? Chiedo scusa, Bruno ha ragione...non ho controllato che il testo fosse aderente alla serie scritta...mi sono fidato ed in effetti questo è accaduto, perché il quesito non l'ho proprio studiato, nella fretta di mettervi al lavoro.....
Va bene, il quesito è da intendersi annullato.
_________________

$\text { }$ciao Immagine ciao
E' la somma che fa il totale (Totò)

Admin
Amministratore del sito
Amministratore del sito
Messaggi: 869
Iscritto il: mer apr 20, 2005 3:47 pm
Località: Benevento

Messaggio da Admin »

Chiudo il topic?
Pietro Vitelli (Amministratore del Forum)
"Un matematico è una macchina che converte caffè in teoremi" Paul Erdös
www.pvitelli.net

panurgo
Livello 9
Livello 9
Messaggi: 1521
Iscritto il: sab nov 19, 2005 3:45 pm
Località: Padova

Messaggio da panurgo »

Sarebbe un peccato, perché nella formulazione di Bruno mi pare interessante: la serie è

$S = 1 + \frac{1}{2} + \frac{1}{3} + \frac{1}{4} + \frac{1}{6} + \frac{1}{8} + \frac{1}{9} + \frac{1}{{12}} + \frac{1}{{16}} + \cdots = \sum\nolimits_{\alpha ,\beta \in N} {\frac{1}{{2^\alpha 3^\beta }}}$

E’ possibile raggruppare i termini in

$S_{0,0} = 1$

e infinite somme del tipo

$S_{n,m} = \frac{1}{{2^n 3^m }} + \frac{1}{{2^{2n} 3^{2m} }} + \frac{1}{{2^{3n} 3^{3m} }} + \cdots$

nelle quali $n$ e $m$ devono essere primi tra loro. Ciò perché se, per esempio, $n= p m$ allora

$\frac{1}{{2^n 3^m }} = \frac{1}{{\left( {2^1 3^p } \right)^n }}$

e quindi sarebbe un termine della somma $S_{1,p}$. Ugualmente per $m = q n$

Consideriamo la somma $S_{1,0}$

$S_{1,0} = \frac{1}{2} + \frac{1}{4} + \frac{1}{8} + \frac{1}{{16}} + \cdots$

Si tratta della serie geometrica di ragione $\frac 1 2$ senza il primo termine, cioè

$S_{1,0} = \frac{1}{2} + \frac{1}{4} + \frac{1}{8} + \frac{1}{{16}} + \cdots = 1 + \frac{1}{2} + \frac{1}{4} + \frac{1}{8} + \frac{1}{{16}} + \cdots - 1 = \frac{1}{{1 - \frac{1}{2}}} - 1 = 1$

La somma somma $S_{0,1}$ vale

$S_{0,1} = \frac{1}{3} + \frac{1}{9} + \frac{1}{{27}} + \frac{1}{{81}} + \cdots = 1 + \frac{1}{3} + \frac{1}{9} + \frac{1}{{27}} + \frac{1}{{81}} + \cdots - 1 = \frac{1}{{1 - \frac{1}{3}}} - 1 = \frac{1}{2}$

La somma somma $S_{1,1}$ vale

$S_{1,1} = \frac{1}{6} + \frac{1}{{36}} + \frac{1}{{216}} + \frac{1}{{1296}} + \cdots = \frac{1}{5}$

E, in generale, la somma $S_{n,m}$ vale

$S_{n,m} = \frac{1}{{2^n 3^m }} + \frac{1}{{2^{2n} 3^{2m} }} + \frac{1}{{2^{3n} 3^{3m} }} + \cdots = \frac{1}{{2^n 3^m - 1}}$

Nella formulazione di Pasquale, la serie è

$S = 1 + \frac{1}{2} + \frac{1}{3} + \frac{1}{4} + \frac{1}{6} + \frac{1}{8} + \frac{1}{9} + \frac{1}{{10}} + \frac{1}{{12}} + \frac{1}{{14}} + \frac{1}{{15}} + \frac{1}{{16}} + \cdots$

ovvero

$S = 1 + \frac{1}{2} + \frac{1}{4} + \frac{1}{6} + \frac{1}{8} + \frac{1}{{10}} + \frac{1}{{12}} + \cdots + \frac{1}{3} + \frac{1}{6} + \frac{1}{9} + \frac{1}{{12}} + \frac{1}{{15}} + \cdots - \frac{1}{6} - \frac{1}{{12}} - \frac{1}{{18}} - \cdots$

e, quindi

$S = 1 + \left( {\frac{1}{2} + \frac{1}{3} - \frac{1}{6}} \right)\left( {1 + \frac{1}{2} + \frac{1}{3} + \frac{1}{4} + \frac{1}{5} + \frac{1}{6} + \cdots } \right) = 1 + \frac{2}{3}\left( {1 + \frac{1}{2} + \frac{1}{3} + \frac{1}{4} + \frac{1}{5} + \frac{1}{6} + \cdots } \right) = \infty$

La serie diverge
Ultima modifica di panurgo il gio dic 29, 2005 8:35 am, modificato 1 volta in totale.
il panurgo

Principio di Relatività: $\mathbb{m} \not \to \mathbb{M} \, \Longleftrightarrow \, \mathbb{M} \not \to \mathbb{m}$
"Se la montagna non va a Maometto, Maometto NON va alla montagna"

panurgo
Livello 9
Livello 9
Messaggi: 1521
Iscritto il: sab nov 19, 2005 3:45 pm
Località: Padova

Messaggio da panurgo »

panurgo ha scritto:$n$ e $m$ devono essere primi tra loro
In realtà, non devono avere fattori comuni, ovvero $MCD = 1$. Infatti, se $n = p a$ e $m = q a$ allora $\frac 1 {2^n 3^m} = \frac 1 {\left ( 2^p 3^q\right )^a}$ e quindi è un termine della somma $S_{p,q}$
il panurgo

Principio di Relatività: $\mathbb{m} \not \to \mathbb{M} \, \Longleftrightarrow \, \mathbb{M} \not \to \mathbb{m}$
"Se la montagna non va a Maometto, Maometto NON va alla montagna"

Ospite

Messaggio da Ospite »

A me il quesito sembra molto semplice!
La somma è 3 infatti essa si può scrivere nel seguente modo:
$(1/2^0+1/2^1+1/2^2+1/2^3+...+1/2^n)*(1/3^0+1/3^1+1/3^2+1/3^3+...+1/3^n)$
Essendo il prodotto di due serie geometriche di ragioni 1/2 e 1/3 si ottiene:
$[1/(1-1/2)]]*[1/(1-1/3)]=2*3/2=3$

panurgo
Livello 9
Livello 9
Messaggi: 1521
Iscritto il: sab nov 19, 2005 3:45 pm
Località: Padova

Messaggio da panurgo »

Ospite ha scritto:A me il quesito sembra molto semplice!
La somma è 3 infatti essa si può scrivere nel seguente modo:

$S = \left ( \frac 1 {2^0} + \frac 1 {2^1} + \frac 1 {2^2} + \frac 1 {2^3} + \cdots + \frac 1 {2^n} + \cdots \right) \times \left ( \frac 1 {3^0} + \frac 1 {3^1} + \frac 1 {3^2} + \frac 1 {3^3} + \cdots + \frac 1 {3^n} + \cdots \right)$

Essendo il prodotto di due serie geometriche di ragioni $\frac 1 2$ e $\frac 1 3$ si ottiene:

$\frac 1 {1 - \frac 1 2} \times \frac 1 {1 - \frac 1 3} = 2 \times \frac 3 2 = 3$
Col che Ospite ha dimostrato anche quanto vale il mio acume... :cry:
il panurgo

Principio di Relatività: $\mathbb{m} \not \to \mathbb{M} \, \Longleftrightarrow \, \mathbb{M} \not \to \mathbb{m}$
"Se la montagna non va a Maometto, Maometto NON va alla montagna"

Bruno
Livello 10
Livello 10
Messaggi: 2020
Iscritto il: lun nov 21, 2005 6:07 pm
Località: Bologna

Messaggio da Bruno »

...

Il tuo acume ha valore alto e intatto!
D'altra parte, la questione è nata un po' zoppicante...

;) Bruno

Rispondi